Questions and Answers about Cosmology

1. Due to the big bang, would this just mean that the universe is still so young that it hasn’t started to slow yet or is there another theory?

No - even if it hasn't started to slow down yet, that is different from actually speeding up.

The standard big bang theory has a clear expectation for how the universe should expand when young with its mass-energy content dominated by ordinary matter.  This period, called "matter domination" begins a mere few hundred thousand years after the big bang itself.  At that time, the expansion would not have slowed down perceptibly as it should have by today if the universe remains matter dominated.  So early on, it wouldn't obviously be decelerating, but neither would it be accelerating.  For that to happen requires an extra push from something very new - the dark energy.  In the very early universe (when it was only 10^-35 seconds old!) it is possible that there was a period of acceleration called Inflation.  That would, in many ways, be analogous to the acceleration we seem to be seeing today, but would have to be a separate event.  The expansion of the universe between then and now does appear to have experienced some deceleration before starting to accelerate again.

Professor Stacy McGaugh

2. Also, is the acceleration of the universe essential to the universe's survival in that it would collapse quickly if it started to slow?

No, the acceleration is not essential to the survival of the universe.  It is quite possible, in the context of Einstein's theory of General Relativity, to have a universe that decelerates so slowly that it is never in danger of re-collapsing.

Stacy McGaugh

3. What would happen if the universe did collapse?

You got me! It is certainly possible for the universe to be so dense that gravity would be strong enough to make it re-collapse.  In this case we would observe blue shifts instead of red shifts as all the galaxies ran back together.  They would collide, and the universe would become progressively hotter and denser.  In the last minutes, even super-dense objects like white dwarfs and neutron stars would begin to evaporate.  Ultimately, the fate of such a universe would seem inevitably to lead to a Big Crunch where all matter is jammed together in one cosmic black hole.  But who really knows?  There has been speculation that after such a Big Crunch there could be a new Big Bang, so that the universe might regenerate in cycles like the legendary Pheonix.  I find that a nice thought, but a big crunch does not loom soon in our future, if it'll ever happen at all:  it won't if our current measurement and understanding of the acceleration is correct.  This universe will expand forever, becoming ever sparser than it already is.

Stacy McGaugh

4. What is dark energy in simple terms?

Dark energy is one of the most fun and fundamental aspects of current-day cosmology.  My answer may seem a little vague, but that's the current state of things, unfortunately.

  Perhaps you've heard of the rubber sheet analogy for spacetime and the effect that mass and gravity has on it?  Imagine putting a bowling ball on a rubber sheet.  It bends the sheet downward, so that a ping-pong ball rolling nearby will be deflected.  If you put another bowling ball somewhere else, it makes its own dent and tends to roll towards the first bowling ball.  This is the way that Einstein thought of gravity.

  But now think of that rubber sheet again, and suppose that something pushes the sheet up.  Now, ping-pong balls will roll away from each other.  That, in a nutshell, is dark energy.  It is thought that an energy field of some sort (whose origin is a mystery!) pushes things apart, causing the universe to expand faster than it would have otherwise.  This pushing is extremely tiny, so you'd never notice it in everyday life, in the solar system, or even in our galaxy, but on the scale of the universe it's important.  People see evidence (from distant supernovae and the background radiation) that the universe may be accelerating in its expansion, so they've begun to take dark energy as a serious possibility.  Incidentally, the first form in which this idea was introduced was as a "cosmological constant" by Einstein. Ironically, he introduced it because he thought it would allow the universe to *not* expand or contract (since observations at that time made the universe seem static), and later regretted introducing it, calling it the greatest blunder of his career.  We should all be able to make such mistakes!

Cole Miller

5. We are in a solar system, and our solar system is in a galaxy, and our galaxy is in the universe. Well do you think that our universe is inside something bigger?

The part of the universe we can actually observe is fixed by the distance light has been able to travel since everything became transparent soon after the Big Bang.  There is probably more outside this region, but the only known way to find out what it is like is to wait for light to reach us from further away. Some people have suggested the whole universe might be one of many quantum fluctuations in something bigger.  In my opinion this is just a rather far-fetched idea so far.

Neal Turner

6. Is there a center to the Universe?

Strange as it may seem, the universe does not have a center (so "where" and "what is it made of" are not meaningful questions).  It is, of course, hard to think about anything that occupies space and yet does not have a center, but the commonest way of thinking about it is this:

Take an ordinary balloon and blow it up a bit.  Use a black marker to put some dots scattered around at random on the balloon.  Now, notice that none of those dots is at the center of the balloon, and if you lived on one of them, you could perhaps travel around the surface of the balloon and visit other spots, but you would not find any center.  And, if you continue blowing up the balloon (no, don't make it pop yet!), each spot will get further away from every other spot, and yet there is still no center anywhere on the surface of the balloon that the spots are all moving away from.

The surface of the balloon is a two-dimensional surface in three dimensional space (and you can only find a center if you are free to travel in that third dimension - you can't find a center for the earth, either except by digging).  The universe is a three-dimensional surface in four dimensional space-time, and the only thing that resembles a "center" is the time at which the expansion started.  And at that time, all the (three dimensional) space of the universe and all the matter that was eventually going to make up the galaxies and stars and planets and you was crammed very tightly together.

Prof. V. Trimble

7. What is Dark matter and do you believe in it?

Astronomers can estimate the mass of a galaxy by measuring the amount of light it emits.  Generally, the more light, the more massive the galaxy. Astronomers can also estimate the mass of a galaxy by measuring the speeds of stars as they orbit the galaxy center.  When we compare the mass derived from the light with the mass derived from orbital speeds, we notice a mass discrepancy:  the mass derived from orbital speeds is higher than that derived from light.  In other words, the stars are moving much faster than expected based on the luminous mass in the galaxy.

The most important thing that you should get out of all this is the following:   The laws of gravity as we currently understand them tell us that there is more mass present in galaxies than what we can see.  The second most important thing is that there are two logical conclusions:  1) There must be a significant amount of matter (dark matter) that emits no (or very little) electromagnetic radiation or 2) The laws of gravity which we use to infer the existence of mass discrepancies are themselves incorrect.  If conclusions #2 is correct, then there might not be any need to invoke dark matter because the mass discrepancies would disappear when we use the alternate gravitational laws (i.e. the mass that we see may be all that there is).  Some astronomers and physicists are currently researching what the alternate gravitational laws might be.  However, most astronomers and physicists prefer conclusion #1 because the laws of gravity we use currently are so well verified by experiments.

If we accept conclusion #1, then we can go ahead and try to figure out what dark matter is.  Mathematical analyses of the rotations of spiral galaxies suggest that the dark matter is located in large halos surrounding the luminous parts of the galaxies.  Dark matter is thought to be composed nearly entirely of either MACHOs or WIMPs.  MACHOs (MAssive Compact Halo Objects) are either brown dwarfs, white dwarfs, or extremely massive black holes.  Various experiments are in progress to try to identify such objects.  WIMPs (Weakly Interacting Massive Particles) are special subatomic particles that are predicted theoretically.  They rarely interact with normal matter (i.e. the stuff that stars are made of) except through their gravitational effects.  If WIMPs do exist, then some of them should be passing through our physics laboratories right now.  Indeed, there are experiments under way that are trying to detect them when they collide with atomic nuclei.

Mike Barker

8. First off, I don't really understand how, or in what way the universe is expanding.  One thing that I do understand is that we wouldn't ever be able to get to the edge of the universe, because the rate at which we travel as humans isn't fast enough.  But my question is, what if there was a way to go faster than the speed of light, would we then be able to get to the edge then?  If not that would we be able to get to stars, or galaxies millions of light years away?  Do you think that there is any possibility what so-ever of a time machine, or something that could send us moving faster than the speed of light?

First some background:

By the late 1920's two astronomers (Humason and Hubble) were photographing spectra of faint galaxies and measuring the distances of these galaxies. When they compared the distances and recession velocities of remote galaxies, Hubble and Humason found that distance and velocity were proportional to each other and that the most distant galaxies had the highest velocities of recession.  This relationship, now known as the Hubble law, can be written as an equation:

v = H * d,

where v is the recession speed, d is the distance, and H is a number called the Hubble constant.

The fact that galaxies obey the Hubble law shows beyond doubt that the universe is expanding uniformly.  A uniformly expanding universe--that is, one that is expanding at the same rate everywhere--requires that we and all other observers within it, no matter where they are located, must observe proportionality between the recession velocities and distances of remote galaxies.

Here is an analogy that may help you see why.  Imagine a ruler made of flexible rubber, with the usual lines marked off at each centimeter.  Now suppose someone with strong arms grabs each end of the ruler and slowly stretches it, so that, say, it doubles in length in 1 minute.  Consider an intelligent ant sitting on the mark at 2 cm.  This ant measures how fast other ants, sitting at the 4, 7, and 12 cm marks, move away from him as the ruler stretches.  The one at 4 cm, originally 2 cm away, has doubled its distance; it has moved 2 cm/min.  Similarly, the ones at 7 cm and 12 cm, which were originally 5 and 10 cm distant, have had to move away at 5 and 10 cm/min, respectively, to reach their current distances of 10 cm and 20 cm.  All ants, like galaxies, move at speeds proportional to their distance.

There is no edge to the universe because that would be a special place which would violate the cosmological principle.  This principle states that the universe is isotropic and homogeneous--the same in all directions and at all distances.  In other words, the universe is about the same everywhere on large scales--there are no special places. There is a great deal of observational evidence in support of this principle.  Therefore, it is the starting point for nearly all theories of cosmology.

The laws of physics as we currently understand them do not allow any object to travel in the universe faster than light. There are, however, theoretical objects called wormholes that could be used to travel between two distant points faster than light without violating the laws of physics. A wormhole has two entrances called "mouths," one (for example) near Earth, and the other (for example) in orbit around the star Vega, 26 light-years away.  The mouths are connected to each other by a tunnel through hyperspace that might be only a kilometer long.  If we enter the near-Earth mouth, we find ourselves in the tunnel.  By traveling just one kilometer down the tunnel we reach the other mouth and emerge near Vega, 26 light-years away as measured in the external Universe.  But wormholes require exotic energy to stay open.  Exotic energy is a special kind of energy that is *negative* as measured by light rays.  Currently, we do not yet know for sure of any sources of exotic energy that would hold a wormhole open.  We need to know the laws of quantum gravity to solve this problem.

A pair of wormholes in motion relative to each other could be used as a time machine.  But theoretical calculations *suggest* that every time machine is likely to self destruct at the moment one tries to create it. However, we cannot know for sure until we have an excellent understanding of the laws of quantum gravity.

Mike Barker

9. In the Case of dark matter why does it have to exist? If the big bang is the cause of our universe's creation it is basically a giant explosion correct. Well in some explosion isn't here a hesitation then a sudden burst of more power? Why does dark matter have to exist?

In some ways, it may seem odd for astronomers to believe in dark matter, since, by definition, it's dark--that is, it doesn't emit any light that we've been able to detect so far.  And, generally speaking, astronomers need an object to give off light for us to be able to figure out its properties.

However, there's a growing amount of evidence that seems to point to the existence of dark matter, regardless of the fact that we can't see it. For example, we can observe how fast stars and gas orbit around the center of a galaxy.  Given the amount of stars and gas that we can see, we can also predict how fast they SHOULD orbit.  These numbers--what we see and what we predict--don't match.  This suggests that there's some other kind of matter that is also present but that we can't see--dark matter. The same kind of thing can also be done in clusters of galaxies, by looking at the speeds of individual galaxies and comparing that to what we would expect given what we can see. As to the "Big Bang": an explosion might be a good way to think of it. I don't think there was any hesitation, though--it would take a lot to stop an explosion like that!

Dave Rupke

10. I was wondering if they will ever build something that can pick up dark matter?

By "pick up" dark matter, I'm going to assume that you mean "see", not "get," since, generally speaking, we'd probably need to see it first.

Anyway, astronomers are definitely looking for dark matter.  And I would guess that we'll eventually find it, assuming that it actually exists. One possible explanation for dark matter is a bunch of really faint normal stuff that we just can't see well (like dim stars or black holes).  People have looked for things like that, but haven't found very much of it yet, at least not enough.  Dark matter might also be some very strange stuff, like new particles or other universes or extra dimensions.  There are some ideas of what the new particles might be, and they might someday be detected in things like particle accelerators.  People are also building some experiments now to specifically look for these dark matter particles, but have yet to turn up anything conclusive.  Explanations like other universes or extra dimensions will probably have to wait longer to be demonstrated . . .

Dave Rupke

11. As an astronomer do you yourself think the universe will expand forever? Toward what specific point do you think the universe will each?  After you answer my questions would you send this to other astronomers, so I can receive variations of opinions?  Thank you for your time.  I think we're lucky to speak with professionals.

That's a great question!  The issue of the fate of the universe has occupied the attention of astronomers for centuries, and it's exciting that just now we're approaching the point where we could have a definitive answer.  It would be too complicated to survey astronomers for their opinions, but how about if I summarize some of what people have discussed? Ever since the discovery by Hubble (1929) that the universe is expanding, people have realized that a crucial question is whether the gravity of the universe is enough to slow down the expansion and eventually reverse it.  An analogy to think of is escape velocity. If you take a ball and throw it up into the air, it will reach some height and then fall down.  If you throw it faster, it will go higher before falling.  In principle, if you didn't have to worry about our atmosphere and could throw the ball about 7 miles per second, you could throw the ball completely off the Earth; that's the escape velocity.  The stronger the gravity (in other words, the more matter you have), the faster you'd have to go.  Put in terms of the universe, if there is enough matter the universe will eventually slow down, stop, reverse itself, and come back together in a "Big Crunch".  If there isn't, the universe will expand forever.

A few years ago, though, people got a surprise when it was discovered that the expansion of the universe is *accelerating*!  That would be like throwing a ball up and finding that it went faster as it got higher :).  Strange behavior.  The root cause of this is suspected to be a field throughout the universe called a "cosmological constant". However, the nature and origin of this field is not well understood. The net result is that although most astronomers think that the universe will expand forever, powered by this field, it isn't well enough understood to say with certainty.  Depending on the exact origin of this field it could even be that the universe expands and contracts in a cyclic way, a kind of "oscillating" universe.  My own personal belief is that the universe probably doesn't do that, and instead is likely to just expand forever.  However, I should point out that the great thing about science is that we don't just have to argue about our opinions!  Ultimately, better observations and more data will tell us what's really going on.  Cosmology is a field that is now entering a phase of data richness, where many ideas will be tested.  Stay tuned!

Cole Miller

12. Recently in Astronomy class we have discussed in little detail the theory of dark energy.  If you could I would like for you or one of your colleagues to help explain the hypothesis' behind this theory.

Dark energy, eh?  I think the shortest, and most honest, answer is - heck if any of us know!  I will also give a long answer.  This will be much longer and less clear, because it is a good, if rather broad, question which gets at much of our current ignorance about the universe.  The idea of dark energy is that the vacuum of space - emptiness itself - has some non-zero energy associated with it.  This is mass-energy in the sense of Einstein's E=mc^2, and therefore affects the gravitational behavior of the whole universe, which is of course filled with vacuum.  It is hard to conceive of how emptiness can have some finite mass-energy.  By definition, there is no stuff there.  If I may borrow an overused phrase from science fiction, it is the fabric of space-time itself with which the dark energy is associated.  That we call it dark energy is, I think, an expression of the human need to label that which we don't understand.  Dark energy is really just a modern name which stands in for Einstein's cosmological constant.  This was a fudge factor he added to his equations of general relativity in the 19teens in order to make a static universe.  His original theory did not allow for this - it said the universe had to be either expanding or contracting - it could not sit still.  This seemed philosophically unacceptable at the time - the universe had to have been there forever. Hadn't it?  Well, no.

In the 1920's, Edwin Hubble discovered that galaxies appeared to be racing away from each other:  the universe WAS expanding, and had a finite age.  After this, Einstein called his cosmological constant his greatest blunder.  If he had been a fan of The Simpson's, he would have uttered a big D'OH! because he should have had faith in his original theory and PREDICTED the expansion.  There is another, less well known way in which the cosmological constant might be considered a blunder.  Einstein put it there to force the universe to be static.  However, it had to have a very special, finely-tuned value to give a static solution.  All other values would make the universe not only expand, but would accelerate that expansion.  Worse, even the special static value was unstable.  Given even the slightest nudge, it too would start an accelerating expansion.  So the cosmological constant did not have the desired effect - it made the expansion faster rather than taming it to a stand still.  This was a mathematical blunder by arguably the most brilliant theoretical physicist of the past century.

Despite this sordid history, there are now multiple lines of modern evidence which suggest that the expansion of the universe is accelerating. For this to happen in Einstein's theory requires a term just like his cosmological constant.  This is what Dark Energy is - an add-on to General Relativity which describes the gravitational effects of there being energy in the nothingness of the vacuum of space.  If you find that a bit hard to swallow, you're not alone.  There are many optimistic cosmologists who say the data require dark energy and are sure they'll figure out what that means some day.  There are a few more pessimistic ones who fear it is still just a fudge factor which covers up what we still don't understand about the universe.

Well, that is a mouthful, and no mistake.

But I hope it makes a good story, even if it may not make Dark Energy any less mysterious.

Professor Stacy McGaugh

13. I would like to know what knew theories have accumulated about the existence of more than one universe? In addition, could we even call it a Universe, (since uni means single)?

Sometimes people refer to the idea of more than one universe as a "multiverse."  Mostly this is speculation, because the laws of physics (as we understand them today) prevent us from ever knowing about other universes.  However, scientists do think about these issues.  Basically, we currently do not understand the very first instant of the creation of the Universe (the "Big Bang") well enough to say whether our Universe is unique.  It may be that the conditions that led to the Big Bang did exist elsewhere (else when?), except the concepts of elsewhere and else when are not really well defined at that first instant.  This is a failing of our current theories.  Until the "unification" of gravity with the other three fundamental forces of nature, this will remain a very speculative topic.

Even then it may turn out that the first instant is unknowable, and so too will be the existence of other universes. Hope that helps! (cosmology is always hard on the brain) :)

-Derek Richardson

14. They say the universe is expanding.  Well, where is it expanding to?  I can't quite grasp the concept that you can get something (space) out of nothing.  Following the theory of multiverses, if our universe is expanding, are the other universes contracting?  Is it possible that there is one sphere-shaped universe that has no outside to it?

You have asked a very good question (and one that I was often asked by college seniors when I taught cosmology recently!).  The basic answer to your question is that the universe isn't expanding into anything.... its simple getting bigger.  It is best to think of a simple analogy.  Suppose that we lived on the surface of a planet that was gradually growing larger.  So the surface area of the planet, i.e. amount of land area, would be expanding.  Its not the case that there is "unused" land that the planet's surface is expanding into... the surface is simply expanding.

Now, you might argue in this case that the planet itself is expanding into previously empty space, and you would be correct.  But you only see that fact when you "step-back" away from the 2-dimensional surface of the planet and view the planet as a 3-dimensional object.  When it comes to the whole universe (which is 4-dimensional, 3 space dimensions + time), there is no other dimension in which we can "step-back" into in order to see the universe's expansion.   All we can perceive is that the universe is getting bigger and bigger, in just the same way that we could see the surface area of an expanding planet getting bigger and bigger.

Chris Reynolds

15. I have always been wondering if there is any intergalactic objects in space, Mr. E has gone over some of it but I don’t understand if there really is intergalactic in space.

There are certainly objects in intergalactic space.  We actually observe x-rays coming from hot electrons that exists in the space between galaxies.  Also, there can be stars (or other objects) that have been "kicked out" of galaxies in intergalactic space.  The complex gravitational interaction between objects can sometimes cause the ejection of stars into intergalactic space.

Rahul Shetty

16. If dark matter is a larger percentage than what we can see, understand, and perceive, then how does it work, and how do we know it's there? How does it influence our lives if we are aware or unaware of its presence?

 We infer that dark matter exists because we perceive its gravitational influence.  Even though we can not see it directly, the stuff we can see responds to it through the force of gravity.  So we see stars orbiting around in galaxies much faster than they should be if the only mass that is there is that in the stars and gas we can see.  These observed motions lead us to infer the presence of extra mass which we can not see - the dark matter.

 There is a great deal of astronomical evidence now that points in this direction:  the rotation rates of spiral galaxies, the bending of light by gravitational lenses, the velocities of individual galaxies within rich clusters of galaxies, and many others.  Some of us consider this conclusive evidence that dark matter must exist.  Strictly speaking though, what we have evidence for are mass discrepancies - either there is dark matter, or we are making some mistake in interpreting the observed motions.  A really conclusive proof of the existence of dark matter would be a laboratory detection of dark matter particles.  If the stuff pervades the universe, we ought to be able to figure out a way to lay our hands on some!  There are, in fact, many experiments going on right now which are trying to do just this.

 The influence of dark matter on our lives is not direct, but does impact our philosophy.  A universe made of stars seems a very different place from one made of invisible matter where stars (and planets and people) are an afterthought of an afterthought.  Though most of us consider it unlikely, there does remain the possibility that the excess motions are cause by a failure of our understanding of physical laws (the force of gravity) rather than the existence of dark matter.  If such a thing were to prove true, it would have an impact on how we view the universe rather like that Copernicus had when he suggested maybe the earth went around the sun and not vice-versa.  No practical implication at all - but very important all the same!

You can read more about dark matter searches at  http://cdms.berkeley.edu/Education/index.html One possible alternative to dark matter is discussed at

 http://www.astro.umd.edu/~ssm/mond/

Professor Stacy McGaugh

17. I have a question about the Big Bang;  according to this theory, the universe keeps expanding.  Does it stand to reason that there is a certain capacity, and once it is reacher, the universe will start contracting?

 The universe is a big place - possibly infinite - and some of our common sense ideas don't apply.  There is no maximum capacity per se, and the universe does not expand into new space:  the same space expands.  That space can be infinite and yet still get bigger all the time! It is possible in principle for the expansion to be halted and for the universe to contract.  This should happen if there is a large enough mass density in the universe that the mutual gravity of all that mass is strong enough to pull things back together.  As best we can tell at present, there is not enough mass to accomplish this, and our universe will expand forever.  The only way to be sure though is to come back and check on things in ten or twenty billion years or more!

Professor Stacy McGaugh

18. I would like to know what would happen if you stepped of the "edge" of the universe. If you could explain this to me in terms that a high school student can understand I would greatly appreciate it.

 People have always wondered about the what-is-beyond-the-edge.  In ancient times, many people believed the Earth was flat, and you would fall off if you stepped over the edge.  I'm sure we can all think of people whom we would like to volunteer for that experiment.  In modern cosmology, there is no physical edge in this sense.  There is a limit to our perception caused by the finite age of the universe.  We can only see out as far as light has had time to travel to us.  The universe is about 13 billion years old, so there is an edge - a horizon - limiting how far we can see.  The most distant thing we could hope to see would be an object 13 billion light-years away.  Light from more distant objects has not had enough time to get to us.   This is not an edge you can imagine stepping beyond, though.  It is more like a horizon.  If you walk far enough on the earth, you can get to and pass the horizon of your starting point.  Nothing special happens - you just can't see back to where you started anymore because you've gone so far that it is beyond your new horizon.  So it is with the universe - you can imagine leaping from galaxy to galaxy until you come to what you had perceived as the edge of the universe.  But all you would find is still more galaxies beyond that horizon, looking very much like the one you left behind.  Your starting point would now be on the edge of your new horizon, but you would not have come to any edge to leap off of.

Professor Stacy McGaugh

19. A) According to the Big Bang Theory, space is continually expanding. If we have came close to the edge of the universe, into what is space expanding? Also, if we were in fact created from the Big Bang Theory, isn't it fathomable that other forms of humans or space are also being or have been created? B) My question is: What is beyond the edge of the universe? And if I were to stand on the edge of the universe and throw a rock out beyond the edge, what would happen? Would the rock still be a part of the universe? Or would it become apart of something else? C)If I were to stand on the edge of the universe and throw a rock off, where would it go?  What would happen to it?

When considering questions of cosmology and the large scale structure of the universe, it may not be surprising to know that one's everyday intuition doesn't always work.  In this case, the difference is that there really isn't an edge to the universe.  Think about a balloon that is being blown up, and imagine an ant on the surface.  The ant can move around the balloon and never find an edge!  If the ant were to throw a bread crumb (in analogy to a rock), it would simply travel along the surface of the balloon.  "But wait", you may ask, "isn't there a center to the balloon?"  Yes, there is.  Here, the analogy is that although the universe doesn't have an edge, it does have a beginning.  You can therefore ask how old the universe is, and the best evidence is that it is a shade less than 14 billion years old.

Beyond the expansion of the universe (our galaxy itself isn't expanding, it is just being carried along), we're not sure.  The question of what our universe is expanding into is a pretty profound one.  Some people think that our universe is expanding into some infinite region, or some higher dimension.  Some think that we can "unask" the question by saying that the universe creates space as it expands, so that it doesn't have to expand into anything.  No one is sure at this time.

It is also an open question about whether there are other universes, and if so what might be in those universes.  Max Tegmark had an article in Scientific American about a year ago that talked about what might happen if the universe were infinite.  He considers different categories, including universes with the same physical laws as ours, and universes with potentially totally different laws.  There are some bizarre conclusions, such as that if the universe is infinite and all the laws are the same, there will be some other universe with an Ashley, Kimberly, and Jessie just like you having lived exactly your life!  You shouldn't take this too seriously at the moment, because although it's fun speculation we don't know how to test those ideas.

Cole Miller

20. What is the matter called outside the universe?

There isn't a special name for matter outside the universe, probably because we don't know that there is any matter outside our universe! As I indicated in an answer to another of your questions, people have speculated about other universes, but it isn't known whether they exist, or what their properties would be.  Therefore, if you want to think about this, you can invent your own names: "hypermatter", "extra-universal matter", or "does it matter" :) :).

Cole Miller

21. Where does dark energy and dark matter come from?

The short and honest answer to your question is that no one knows for sure what dark energy and dark matter are, or where they come from.  Their existence is inferred based on astronomical observations. The best current guess is that dark matter consists of subatomic particles; the tricky bit is that you can show that those particles have to be of a type not yet detected in any experiments on Earth. There are people working on trying to detect dark matter in various ways, so if that is successful then we may know more.  Dark energy is even more mysterious.  Some people think that this is effectively a "cosmological constant" field that pushes spacetime apart.  It may be that grand unified theories of particle physics will eventually predict such a thing naturally, in which case the evidence for the correctness of such a theory in other ways (that can be tested!) will be indirect evidence for the nature of dark energy.  However, I can't think of an experiment that would detect dark energy directly!

Cole Miller

22. I was wondering if it is possible to have more then one universe? If so, could the collapse of one or more of these universes be responsible for the expansion of our universe?

First, let me introduce myself.  I'm a professor at the University of Maryland specializing in black holes and neutron stars, so it's fun for me to consider your questions.

It is also an open question about whether there are other universes, and if so what might be in those universes.  Max Tegmark had an article in Scientific American about a year ago that talked about what might happen if the universe were infinite.  He considers different categories, including universes with the same physical laws as ours, and universes with potentially totally different laws.  There are some bizarre conclusions, such as that if the universe is infinite and all the laws are the same, there will be some other universe with someone exactly like you having lived exactly your life!  You shouldn't take this too seriously at the moment, because although it's fun speculation we don't know how to test those ideas.

Getting even weirder, when people have thought about what caused the Big Bang, one concept is that there is really another spatial dimension, and that in this spatial dimension there are four-dimensional "branes" (short for "membranes") that move around.  This idea proposes that the collision of two of these branes collided and produced the Big Bang.  Therefore, yes, people are thinking along the lines you suggest!

Cole Miller

23.  There is evidence of there being more than three dimensions, but my question is this: How would we detect these separate dimensions if there are any? I presume that mathematics would be the basis of the study, but I was wondering if you could give me an explanation that is mentally palatable to me. It would also help me understand several of my other questions of the universe.

  For various theoretical reasons, many people believe that if there are other dimensions, the only force that would operate across those dimensions would be gravity.  Therefore, you wouldn't see things across dimensions, but on very small scales you might feel their gravitational pull.

If that's the case, then the mathematical basis you mention would be the following.  You know that Newton's law of gravity says that the force of gravity is proportional to the inverse square of the distance between objects.  It turns out that this originates from the number of spatial dimensions we have; just like the intensity of light drops off as the inverse square of distance (because the area of a sphere increases by the square of distance), the intensity of gravity drops off like that.  Now suppose that you have four spatial dimensions instead of three.  By analogy, you'd expect gravity to drop off like the inverse cube of distance.

How do you measure all this?  The theoretical idea is that although the three spatial dimensions we're familiar with extend a long way in all directions, a fourth might extend just a little way.  Therefore, in order to see deviations from the inverse square law, you need to make very careful measurements of the gravitational force between objects that are extremely close to each other.  This has been done, and there is no evidence of any change down to separations of 0.0001 of a meter.  Still, people keep trying.

Cole Miller

24. I would like to know why you think the universe is still expanding, could it be that the Big Bang was a lot larger than we think or is there some unexplainable substance pulling on the universe that we just can't find yet.

We are confident that the universe is expanding because if you look at almost any galaxy, you find that it is moving away from us.  In fact, anyone in any galaxy would find the same thing!  The only way to explain this is if the universe is expanding; that is, galaxies tend to move farther away from each other as time goes on.  Therefore, the patch we see is definitely expanding.

What isn't known is whether what we see is all there is.  In fact, it is an open question whether there are other universes and if so what might be in those universes.  Max Tegmark had an article in Scientific American about a year ago that talked about what might happen if the universe were infinite.  He considers different categories, including universes with the same physical laws as ours, and universes with potentially totally different laws.  There are some bizarre conclusions, such as that if the universe is infinite and all the laws are the same, there will be some other universe with someone exactly like you having lived exactly your life!  You shouldn't take this too seriously at the moment, because although it's fun speculation we don't know how to test those ideas.

Cole Miller

25. Mr. Edwards was explaining the Big Bang theory to us and he said it is theorized that at the Big Bang the universe was comprised in just a point. If that is the case, then does that mean that that point was the universe? And do you have any theories about where point originated? And what came before the Big Bang? And doesn't there have to be space for that point to have existed in?    I was always told from the time that I was a little girl that the universe is infinite but I am getting a feeling after being in this class that it is finite. This is driving me crazy and I was hoping that you could better explain this to me in a way that I can understand.

The reason you have a hard time articulating those questions is because your questions are very difficult for even astronomers and cosmologists to understand.  You are not alone!  Unfortunately, I (or any other scientist) may not be able to help you figure out the answers to these questions.  (I hope I don't confuse you any further :-)

According to the Big Bang theory, both time and space themselves were created in the Big Bang.  So, the singularity (the point) from which the Big Bang came contained the entire Universe -- all the space, time, matter and energy that we see today.  Because the entire Universe was caught up in that singularity, we can say that the Big Bang happened *everywhere* at once.

The short answer to what came before the Big Bang and to what caused it is that astronomers and physicists don't really know.

The longer answer is that scientists can't really study what came before the Big Bang or what caused it.  By definition, a scientific theory must be testable.  However, everything that we know about the Universe comes from observing space, time, matter and energy -- all of which came from the Big Bang itself.  There is no way for us to see anything before the Big Bang, or, for that matter, beyond our Universe; therefore, there is no way to test any theories about what came before the Big Bang or what is outside our Universe.

Having said that, there are some cosmologists working on theories that would have implications for what was around before the Universe that we see today. Alan Guth is one such cosmologist, and he wrote a book called "The Inflationary Universe".  If you are really interested in the beginning of the Universe, you might want to check it out -- the book is written for non-scientists, but it might still be a bit of a challenge for you to read.

Barbara Mattson

26. I was wondering how do scientists calculate the average temperature of the universe? As you may know, the average temperature of the universe is approximately 2.73 Kelvins.  (A Kelvin is like a Celsius degree, except that you add 273 to the temperature in Celsius to get the temperature in Kelvins.  So, 2.73 Kelvins is -270.27 degrees Celsius.  At 0 Kelvins, all motion stops - even the motion of the electrons in atoms - and that's the coldest temperature that could exist.)

It might surprise you that this temperature was predicted before it was actually measured.  The radiation that produces this temperature comes from the Big Bang.  The Big Bang was the explosion *of* space (not into space) that began the universe.  This theory was first proposed by George Gamow in the 1940's and is accepted by many people today.  When Gamow proposed his theory, people started to realize that the universe must have begun as very hot.  The theory says that the energy that came out of the Big Bang was in the form of the most energetic photons - gamma rays.  As the universe got older, it expanded and cooled, and so have those photons, which are now much lower energy and longer wavelength.  The theory predicts that today, the radiation would be in the microwave part of the spectrum and correspond with a temperature of 2.73 Kelvins.  As the universe ages even more, the temperature will continue to decrease.

The story of how this was first detected is one of the neat chance discoveries of astronomy.  In the 1960's, Arno Penzias and Robert Wilson, who worked for Bell Telephone Laboratories, were trying to develop ways to relay telephone calls via satellites.  They kept coming across this constant "noise" in their detector every time they pointed their antenna towards the sky.  It turned out that what they were detecting was the 2.73-Kelvin "Cosmic Microwave Background" radiation that had been predicted in the Big Bang theory.  This is one of the best pieces of evidence that the theory is true.

The famous COBE (COsmic Background Explorer) satellite that was launched in 1989 was able to measure the universe's background temperature within the first few minutes of its operation.  It detected - you guessed it - 2.73 Kelvins.  Further, it discovered that the temperature isn't constant all over - there are slight fluctuations throughout space, which are the result of density fluctuations in the matter of the early universe, and could explain how early matter arranged itself.

Katie McGleam

27. What is dark energy?  Is it a single force, or is it to chaotic to classify?

The nature of the dark energy is still an enigma.  The concept of the dark energy is introduced at 1998 to explain the observational fact that the universe is expanding and accelerating. Supernova observations show that the 14-billion-year-old universe has speeded up its expansion rate in the past 7 billion years. Dark energy produce a repulsive force that can account for the acceleration of the universe.

Note that scientists are not a bunch of irresponsible people who just call something they don't know with a new term.  By including a new form of energy called 'dark energy' in the current Big Bang theory mathematically, all current observational data seem fit together very well. It's like at the time Newton developed the theory of the gravity, people may say "what gravity?"  But his theory does explain apples falling on the ground and the Moon rotating around the earth at the same time.  Then when people invent rockets, Newton's theory also correctly predicts rockets' trajectories. Dark energy is a very new and probably revolutionary concept.  It may or may not survive after new evidence is collected, but scientists are optimistic now.

Cole Miller

28. We now are nearly positive that the universe is not infinite, that there is an end to it somewhere. But would this "end" be a definite barrier, or would this edge just kind of fade into nothingness? If the universe was created in a massive explosion, then, like a normal explosion, there would be a sort of wall of fire. But since the universe is around 13.8 billion years old, would this barrier be broken down by now, like an explosion that only lasts for a fraction of a second?

Don't think about the Big Bang as an earthly explosion. Strickly speaking, when we imply that the universe was created in the Big Bang, what we mean is that space and time were created in this event. As the universe expanded (space spreads out), time runs forward.

As astronomers look out to greater and greater distances we are looking back in time since the light from distant objects takes time to move through space.  For example, it takes sunlight 8.3 minutes to travel to the Earth, so we see the Sun as it was 8.3 minutes ago.  A star located 1000 light years away may explode as a supernova. It takes the explosion's light 1000 years to reach Earth--we see it but there is a delay between our observations and the actual event.  A very distant object, a quasar for example, might be located at a distance of 12 billion light years.  We are seeing this object as it was 12 billion years ago--it most likely is a young galaxy with a very energetic nucleus.

Now could we look far enough out to see the beginning of time, to see the Big Bang?  Theoretically, we could (by looking in any direction) look past the very young, forming galaxies (quasars) to a more distant location before any galaxies formed, and looking all the way back in time to the Big Bang event. During the early years of the universe's existence, there would be a lot of energy.  Space would be so hot, that you wouldn't find molecules, or even atoms.  Theory tells us that there would be nuclei (centers of atoms), free electrons moving fast, and photons (light).  Conditions would be too energetic for any atoms to form.  In this state, the electrons and the photons interact.  The net result is that the photons (light) from this early era are scattered (sort of like bouncing around between the electrons). They aren't moving in a straight line, so they can't complete a path to our Earth (eventually located billions of light years away).

About 300,000 years after the Big Bang, the universe cools to the point where atoms can form.  At this time the photons were free to move, and many completed a journey that takes billions of years to Earth.  This energy is detected today, in every direction that we look, and is called the cosmic background radiation. This is as far back in time as we can see--300,000 years after the Big Bang.

This is sort of like the "wall of fire" that you describe, except the radiation (photons) are red shifted, so the energy is very weak when we observe it on Earth.

Grace Deming

29. What is the pull that keeps the universe expanding, or what is a logical explanation?

Well, I think it's easiest to think of the expansion of the universe like this:  Imagine a round bomb exploding.  After it explodes, all the little pieces of the bomb move away from the explosion center really fast.  Now imagine that the bomb is the ENTIRE UNIVERSE.  When the universe began, there was this big explosion (called the "Big Bang").  That explosion sent all the pieces of the universe (namely the galaxies) moving away from the explosion center really fast.  That's what the expansion of the universe is--all the galaxies moving away from each other.  The universe is still expanding, but the rate of expansion is slowing down ever so slightly, because of the gravitational pull of all the galaxies.

I just want to say though, that this explanation is only the currently accepted THEORY. There is no concrete proof that this is actually what is happening.  In fact, there is current research that indicates that the universe is not expanding, but contracting!  So the bottom line is to keep an open mind about this.  Research is always going on to find out the right answer.  That's one thing that makes astronomy so fun--we're trying to find out things that nobody knows!

Carl Gross

30. A) Due to the big bang, would this just mean that the universe is still so young that it hasn’t started to slow yet or is there another theory?

No - even if it hasn't started to slow down yet, that is different from actually speeding up.

The standard big bang theory has a clear expectation for how the universe should expand when young with its mass-energy content dominated by ordinary matter.  This period, called "matter domination" begins a mere few hundred thousand years after the big bang itself.  At that time, the expansion would not have slowed down perceptibly as it should have by today if the universe remains matter dominated.  So early on, it wouldn't obviously be decelerating, but neither would it be accelerating.  For that to happen requires an extra push from something very new - the dark energy.

 In the very early universe (when it was only 10^-35 seconds old!) it is possible that there was a period of acceleration called Inflation.  That would, in many ways, be analogous to the acceleration we seem to be seeing today, but would have to be a separate event.  The expansion of the universe between then and now does appear to have experienced some deceleration before starting to accelerate again.

B) Also, is the acceleration of the universe essential to the universe's survival in that it would collapse quickly if it started to slow?

No, the acceleration is not essential to the survival of the universe. It is quite possible, in the context of Einstein's theory of General Relativity, to have a universe that decelerates so slowly that it is never in danger of recollapsing.

C) THEN, what would happen if the universe did collapse? You got me! It is certainly possible for the universe to be so dense that gravity would be strong enough to make it recollapse.  In this case we would observe blue shifts instead of red shifts as all the galaxies ran back together.  They would collide, and the universe would become progressively hotter and denser.  In the last minutes, even super-dense objects like white dwarfs and neutron stars would begin to evaporate.  Ultimately, the fate of such a universe would seem inevitably to lead to a Big Crunch where all matter is jammed together in one cosmic black hole.  But who really knows?  There has been speculation that after such a Big Crunch there could be a new Big Bang, so that the universe might regenerate in cycles like the legendary Pheonix.  I find that a nice thought, but a big crunch does not loom soon in our future, if it'll ever happen at all:  it won't if our current measurement and understanding of the acceleration is correct.  This universe will expand forever, becoming ever sparser than it already is.

Stacy McGaugh

31. Why is the universe spreading out faster and faster, why?

"The Lambda.  That is the only answer." - Willem de Sitter, early 20th century cosmologist

Indeed, it is hard to give a better answer, even many decades later. Lambda is the symbol used by Einstein to denote his cosmological constant.   Ironically, it was intended to prevent the universe from expanding.   However, it only has that effect for a very, very particular numerical value.  For all others, it causes the expansion of the universe to accelerate, as we now seem to be observing.  Worse, the special value is unstable, like a pencil balanced on its point.  Any tiny nudge will make the universe transition from static to accelerating.  Einstein referred to this as his "greatest blunder" - patching up his theory to force the universe to be static (the philosophical prejudice of the day being that this must be the case) when instead he should have predicted the expansion later observed by Hubble.  Ironically, if he hadn't made this mistake, then we would have no way of explaining the currently observed acceleration.

Today we often call it "dark energy," but this is little more than the Lambda term Einstein originally envisioned (albeit for the wrong reason!). Remember that mass and energy are equivalent.  IF the vacuum of space itself has non-zero substance, it takes the form of an all-pervasive energy equivalent to Einstein's Lambda.  This "substance" has a curious property - it pushes the expansion of the universe to accelerate.  It is sort of the opposite of a rubber band, which must shrink to relax after being stretched.  The acceleration is very counter-intuitive because it seems like the universe is gaining energy from nowhere.  But unlike the rubber band, in order to relax the dark energy must expand further and faster!  This trick is possible by a simple mathematical slight of hand:  we have in effect invented stuff for which the sign of the force of gravity is reversed so that it repels instead of attracts.  This "anti-gravity" smacks of the worst sort of science fiction, and I can't help wondering if we're missing something fundamental, with the dark energy just being a sort of place holder for our ignorance.  So next time you make a sign error on your homework, you might try passing it off as a new theory - that is essentially what we cosmologists are doing with dark energy.   Don't place too much hope in this being an effective long-term solution, though!

B) One day could it possibly be the first thing to travel faster than the speed of light, since it is accelerating?

Well, what thing?  When we say the universe is expanding, we mean that the space in which galaxies are embedded is expanding.  The galaxies themselves are not flying apart as from a big explosion into a pre-existing space.  The space itself is stretching out.  It is rather like making raisin bread, with galaxies as the raisins.  When the dough rises, the distance between the raisins increases.  But the galaxies themselves are not flying away from each other in a way which challenges the speed of light.  Rather the "fabric" of space is being stretched.  No one piece of that fabric is in any danger of ever exceeding the speed of lights.  It is merely the accumulation of many intervening expanding patches that can give remote objects the appearance of moving that fast. This is an illusion - they are not flying apart like the pieces of an explosive; the space between them is merely getting stretched - like the rising bread dough, but it is just empty space that is stretching.

A period of superluminal expansion is widely thought to have occurred in the first moment of the universe, during "Inflation."  We observe the universe (by way of the cosmic microwave background) to be very uniform in temperature when it was young.  Remarkably, patched of the universe on opposite sides of the sky have nearly identical temperatures. These patches are far out of causal contact now - the light travel time between them is much greater than the age of the universe.  But if the expansion went through this early Inflationary phase, in the remote past those patches would have been bunched together, and so in close thermal contact.  They are now far apart exactly because the nothingness of empty spaces expanded "faster" than the speed of light - which it can do, because it is nothing!

Professor Stacy McGaugh

32.  In relating to astronomy I have heard that during the big bang all matter for something like a billionth of a nanosecond was heated to an extremely high temperature in the trillion degrees and was changed into a new type of subatomic particle and then turned back as it cooled.

You're absolutely right!  As you know, the universe is now expanding. If you imagine running it backwards in time that means that the universe was smaller in the past.

Now, think about what happens when you compress something.  For example, if you use a bicycle pump, then the pump warms up.  Part of this is just friction, but it would happen even without any friction because when you compress air it heats up.  Similarly, when the universe was smaller, it was hotter.

If you extend that all the way to when the universe was just a tiny fraction of a second old, then it was extraordinarily hot.  Just as you said, there was therefore a time when the universe had a temperature of many trillions of degrees.  That is so hot that matter was in a completely different form from what it is now.  In particular, now we have atoms, and those atoms are made up of electrons on the outside and protons and neutrons in the nucleus.  Those protons and neutrons are made up of even smaller particles called quarks, and those quarks interact with each other by passing back and forth things called gluons (I'm not making this up; these are real things, but obviously the scientists who named them were having some fun!).

In our everyday experience, it turns out that one can't have any "free" quarks or gluons; they have to be bound together in bigger particles such as protons or neutrons.  However, when things were really hot in the very early universe, the quarks and gluons could go free.  As the universe expanded and therefore cooled, the quarks and gluons got bound together in protons and neutrons.  The earlier phase, though, was called a quark-gluon plasma and it can now be created for a fleeting instant in particle accelerators.

Cole Miller

33. Will we ever get to the point that will be able to see beyond the cosmic microwave background noise?

This is an interesting and important question.  The issue, as you know, is that if one considers only light or other forms of electromagnetic radiation, the universe was opaque before the era in which it produced the background radiation.  Therefore, we can't "see" earlier than that by standard means.  If we want to figure out what happened earlier, we have two options: (1) think about something else that wasn't opaque, (2) make an *indirect* inference about what happened early.  Let's take these in order.

Since electromagnetic radiation is out, what else might come from the early universe?  One possibility is neutrinos.  These are ghostly particles that are produced in some nuclear reactions, and they interact so little that a typical one could go through a light year of solid lead without scattering once.  That's nice for our purposes because they aren't blocked from the early universe. When enormous numbers of them are produced (as in a supernova) or are really nearby (as in the Sun), it is possible to detect them in bulk. It is expected that there is a neutrino background in the universe, in a way analogous to the microwave background.  This background would have come from when the universe was less than a minute old, so if it could be detected we'd learn something about the early universe. However, unfortunately, detecting those neutrinos would be really, really, tough because they have extremely low energy and that means they interact much less than even ordinary neutrinos!

The other possibility along those lines is gravitational radiation. Einstein's theory of general relativity asks us to imagine spacetime as a rubber sheet: putting a massive object on the sheet causes a distortion, and if it moves back and forth then there are ripples produced, and this is gravitational radiation.  Gravitational radiation interacts even less than neutrinos, so much so that it has not yet been detected directly although many experiments are trying.  If we saw the gravitational radiation background from the early universe then we could see back to when the universe was perhaps 10^(-43} seconds(!!!) old.  However, again, it might be impossible to see this with foreseeable technology.  We'll see.

The indirect methods of "seeing" before the cosmic microwave background involve a few things.  First is the abundance of various elements. You may know that the universe is mainly hydrogen (about 75% by mass), with most of the rest being helium.  These were produced in a first few minutes of the universe's lifetime, and the precise ratio of hydrogen to helium (and some other light elements) gives us an indication of the conditions then.  It is also possible to look in detail at the ripples of the cosmic microwave background to get a sense of what things were like much earlier, maybe much less than a trillionth of a second after the Big Bang.  These methods, although indirect, give a pretty decent picture of the very early universe.

Cole Miller

34. If the universe's matter had at one point all been in one giant mass, then wouldn't it of been some kind of black hole at some point in its evolution? That, to me, poses a lot of possibilities, perhaps there was no Big Bang giant mass, or maybe there’s some manner in which a singularity might explode.... I would like to know what the well known scientists know or think about this.

I'd like you to consider the following thought experiment.  Let's think about this in terms of everyday Newtonian gravity.  Suppose the universe were infinite and filled uniformly with matter.   Where would gravity force matter to move?  The answer is that if the matter were really uniform, it would have no more reason to go in one direction than the other, so it wouldn't move.

Now suppose that the density of the matter is large enough that if you measured some in a spherical region, you'd say that there was enough mass in that region to form a black hole.  Would things change? No!  The matter is still being pulled in every direction with the same force, so it won't collapse.

That is the situation with the universe at large.  If the matter is distributed uniformly (as it basically is early in the universe), then you won't get collapse to a black hole because the matter is being pulled everywhere by the same amount.  Therefore there really isn't a contradiction.

However, there is a subtlety.  Having the universe expand means that spacetime itself can expand.  This expansion is a little different than we're used to, and in particular spacetime can expand faster than the speed of light, meaning that if you somehow mark two points in spacetime, then they can recede from each other faster than the speed of light. That sounds like a contradiction of the special theory of relativity (which says that no object can move faster than light), but it isn't. If you work this through very carefully you find that no local observer (that is, someone fixed at a point in spacetime) will ever see something moving faster than light.  In fact, in the final analysis, it's rather boring: if spacetime is expanding from you faster than light, you just don't see that part of spacetime!

Cole Miller

35. Why would dark energy suddenly start pushing our universe after all this time, or has it always been there?  Is the dark energy simply astronomers' current response to why the universe is accelerating or do they know a lot about it?

Well, the idea is that the dark energy has always been there, but only recently (in cosmic terms - a few billion years ago!) did it start to win over the attractive gravity of ordinary matter.  If it is there, then it must eventually win out and make the universe accelerate.  But it is quite a coincidence that we happen to live so close to when this occurs.

I think your second question hits it right on. We do not know a lot about the dark energy - virtually nothing, really. It is simply our first guess as to why the expansion of the universe appears to be accelerating.

Professor Stacy McGaugh

36. Would you be kind enough to expound upon dark energy for I have found it to be an extremely interesting subject.

Dark energy is a hot topic in the press these days.  And with good reason--a number of recent astronomical observations have suggested that, indeed, the universe consists not only of the `normal' matter that you and I are familiar with (what astronomers and physicists call `baryonic' matter), but also two other substances.  These are `dark' matter and dark energy.  The things that astronomers observe directly all give off some form of light (whether visible, infrared, X-ray, or radio `light').  However, we have not yet directly observed dark matter or dark energy directly, but rather only inferred their presence from observing how they affect other things.  They are thus both given the label `dark,' to indicate that we don't observe them directly through their radiation.

One could say that about another class of astronomical objects as well, black holes.  Unlike black holes, dark matter and dark energy are poorly understood, though the implications of their existence are not.

The need for dark energy is actually quite simple, in a way.  There are a rather straightforward set of equations that describe the evolution of the universe on large scales.  To get dark energy, we modify these equations by adding a constant term, sometimes called the `cosmological constant'.  For a long time, this constant was considered the realm of crackpot scientists.  However, recent observations of distant supernovae, galaxy clusters, and other cosmic objects have forced us into bringing the constant back.  (It was originally added to the equations by Einstein, but he often referred to this as his `biggest mistake'!)

In the current theory, dark energy dominates the matter and energy in the universe.  You may know that matter and energy are interchangeable, or equivalent, in Einstein's framework of `special relativity.'  The normal, baryonic matter that you and I know constitutes a few percent of the total matter and energy in the universe; dark energy makes up almost 3/4 of it!  And that's only at the present day.  Sometime in the distant future, the universe will be mostly dark energy.  That's because, while normal matter attracts other things made of normal matter (i.e., gravity), dark energy stretches things apart.

Unfortunately, although our standard picture for the universe requires dark energy, no one really knows what it is.  Physicists and astronomers are trying to understand it by using rather complicated theories (you may have heard of string theory), but the solution is not an easy one. Perhaps it will be solved in our lifetimes . . . the alternative is that our overall picture of the universe requires some modification.  But that would be an interesting prospect, too. Dave Rupke

37. A) I heard that the universe is expanding possibly faster than the speed of light - because of the nothingness... well I was wondering - does that mean that there are no multiverses?  If there are multiverses, is it likely that one has ever crashed into another?  Or that there is a mega-black-hole somewhere in the nothingness...  Also, what would happen if another universe crashed into ours? B) If our universe is expanding without signs of stopping could parts of our universe eventually run into something else?  If so what? C) Is it possible that a star could be so far away that we haven't seen the light from it yet. So it could be billions of light year away, so that the light could have not reached the earth yet, is that a possibility? Thank you for your time in answering my question.

The idea of multiverses is one that is speculative, meaning that people have various opinions but the answers can't be proved one way or the other yet.  For various wild ideas, check out the article by Max Tegmark in Scientific American (May, 2003). One idea is that there is only one cosmos, but that it is infinite and we see only a small part of it.  Imagine, therefore, that the universe is a grid extending indefinitely in all directions, and that we happen to live in a region where we can see only one square of the grid.  Our square is expanding, as are all the others, and we can see more and more of the grid as time goes on, but there is still more out there.  Therefore, to answer Ashley's question, it could be that there are an infinite number of stars that are far enough away that we can't see them! However, since this is a frontier subject, people have suggested other ideas as well.  In particular, there has been discussion that rather than having continuous dimensions (where you can go from any point in space to any other point in space without leaving the space), you have separate "membranes", usually called just "branes".  Imagine having a number of sheets of paper, and ants that are forced to live on a single sheet.  For our purposes, the ants are living in a two dimensional world.  If the sheets of paper are far enough apart from each other, then ants can never go from one sheet to another. For practical purposes, then, the sheets represent separate universes to the ants.  If two sheets get really close to each other, though, the ants might notice some big changes to their universe. The analogy in our case is that people have considered separate "branes" with extra dimensions that move around and mostly don't interact with each other.  Gravity is supposed to act from one brane to another but other forces can't make the jump.  When two branes get close to each other they can collide, and some people have suggested that this is what leads to an event like the Big Bang! Cole Miller

38. Do you think we will ever be able to see dark matter for sure? What do you think dark matter is made of? Thank you for your time.

You raise an important question.  As you know, at this point we have only indirect evidence for dark matter.  For example, galaxies seem to rotate faster than they could if their only matter was visible. However, ultimately, direct detection will be the only way to know for sure. Most people believe that dark matter is likely to be made of some new type of subatomic particle.  There are some specific candidates, but for various reasons it can't be any particle that has yet been discovered.  If it is a new particle, the way to establish this is to detect it directly.  There are some current experiments that are trying, but so far without success. Do I think we will *ever* see dark matter for sure?  Yes, I do, for the simple reason that "ever" is a long time, and we have made so much progress so far in understanding particles that I'm optimistic we'll manage eventually.  It might be a lot tougher, to detect "dark energy", which is thought to be responsible for causing the expansion of the universe to accelerate.  For that, maybe the best hope is that if eventually we have a "theory of everything" that can explain in some simple way all the particles and how they interact, then dark matter and dark energy will be found to be natural consequences of the theory.  We'll see! Cole Miller

39. If the universe was to start to collapse then would it pull every thing into one ball of "stuff" or would there be some things that wouldn't collapse as fast and be left behind? If the universe was like a balloon and we blew it up and put different things inside, then deflated it, everything would still be inside the balloon. So, if the universe collapsed would it work the same way or what??

People believe that if the universe were to recollapse, then everything would collapse together.  To use your balloon analogy, if the balloon collapses, every part of the surface of the balloon comes together: you don't have some piece of the surface staying behind.  So, no escape if there is a collapse!  On the other hand, it appears most likely that the universe will keep expanding forever, so this issue probably won't arise. Cole Miller

40. It's been discovered the universe is accelerating, but according to physics, I thought this couldn't really be since theoretically nothing can travel faster than the speed of light. So then we were discussing that maybe the acceleration of the universe is not because the actual matter is accelerating, but maybe the space between the matter is accelerating.  Would this be considered dark matter...or dark energy? If the space between matter is separating and causing matter to move apart, making the impression that the universe is accelerating, what do you think is filling these separations of space----is it nothing, just like a hole in a balloon? is this dark energy? What theories are popular about this subject now?

Let's approach this first by clarifying what can and cannot move faster than the speed of light.  This is fairly subtle, so I hope I can explain it well. Special relativity says that *as measured in a local frame* no matter or energy can move faster than the speed of light.  That is, if I set up a laboratory in some small region of the universe, and measure the speed of things going across that region, nothing can move faster than light. What *is* allowed is that spacetime *itself* can spread faster than the speed of light.  Therefore, your statement above is exactly right: it is the space between the matter that is accelerating, rather than the matter itself.  For example, suppose that at some time we have two pieces of matter marked with "X":                             X X Now spacetime expands dramatically, as in the idea of "inflation" near the beginning of the universe:   X                                                            X Neither bit of matter has actually been accelerated to greater than the speed of light, but their separation has increased faster than light could travel.  The net result is rather mundane: they simply can't see each other anymore. As far as what fills the space, the most widespread idea is that this is indeed the role of dark energy.  Basically, the concept is that every given volume of space (whether it has matter or not), say every cubic meter, contains a certain amount of dark energy, and the role of the dark energy is to push things apart.  When the universe was small, there wasn't much volume, so there was very little dark energy and therefore the push was negligible.  As the universe has expanded, however, the volume has increased and therefore so has the accelerating effect. Right now, the acceleration effect is greater than gravity.  Note, though, that this is such a frontier issue that we shouldn't be too confident in our projections!

Cole Miller

41. I was wondering that according to the big bang, space itself is expanding. So if space is expanding, into what is it expanding?

The short answer is that people don't know, because all we can say for sure relates to what we can observe directly, and this question asks about things beyond the bounds of the observable universe.  Some people think that space is being created as the universe expands, meaning that it isn't expanding into anything.  Other people think that there is an infinite expanse out there into which we are expanding. Still others believe that the cosmos (the totality of everything) has a larger number of dimensions than we are used to, and that as a result we can think of ourselves as "membranes" on the surface of these higher dimensions.

This is all fun stuff to contemplate, but for the scientific process to work one has to relate these ideas to something observable.  At the moment is seems difficult to understand how to test many of these concepts, but it might be possible to distinguish scenarios based on the strength of gravitational waves produced in the early universe. Unfortunately, it is also possible that detectors will not be able to see these waves, because there are many stronger sources (such as white dwarf binaries) that would mask the signal.  We'll see!

Cole Miller

42. The first question that I have for you is how do the Astronomers know how the universe was created? Yes, Yes I know this may be a stupid question towards you, but I want to learn more about it because Mr. Edwards had not satisfied my hunger of learning when he taught me this subject. The reason why I have this question is because first of all I have wondered how do you know the way it was created. For were you there? That is why I want to know about this question. I thank you for allowing me to send you these questions and being able to ask you questions.

This isn't a stupid question at all! First, we can establish that questions of *ultimate* origins can not be answered. If I give you an answer (e.g., "about 14 billion years ago, there was a giant explosion we call the Big Bang"), you can legitimately ask "why?", and I can't answer. Maybe some day I could give an answer in terms of something more fundamental, but you could then ask "why?" about the new reason. Therefore, we don't know in that sense how the universe was created.

So what *can* we say? What we can say is that the general model that in the past the universe was smaller, hotter, and denser, has been supported by many observations. These include the observed expansion of the universe itself, the afterglow of radiation that we see as the cosmic microwave background, and even the abundance of the lightest elements (things like hydrogen and helium).

What this says is that astronomers know what the universe was like many billions of years ago because they produce hypotheses, figure out what implications those hypotheses have, check them against observations, and find that they work pretty well. That's the scientific method in general. In this case, it works out well enough that people are quite confident that we have the basic features worked out back to just a few minutes after the Big Bang, and pretty confident that we know what happened back to a tiny fraction of a second after the Big Bang.

However, we don't know what caused the Big Bang, if there was something before the Big Bang, or whether there are other universes that were produced at the same time. Various people have different ideas, but because we don't have observations that bear on this we don't know.

Good question!

Cole Miller

43. I wonder if nothing exists, neither life, neither planets, and galaxies, what would be, what would happen and what would exist. That makes me ask, what is the sense of everything?? How could we exist??

What is the size of the universe??? If there is a size, the universe is not a compact and understandable thing, its as wide as the mind of a young boy beggining his life, and he uses his imagination to understand the things, while the mature man has for everything an explanation and definition, thats why when we think about the universe, we think using a children's mind, because we wonder and try to understand where did we come from and all that stuff, that we didn't find an explanation yet, its like a magic show, this make us wonder what is the secret and how did the magician do that, our mind is creative again.

You appear to have two questions: (1) why do we exist, and (2) what is the size of the universe. I'll address the second one first.

The size of the universe can be considered in two ways. The volume that *we can see* is fairly definite. We can currently see out to about 13.7 billion light years, because the universe is 13.7 billion years old. We can measure distances in various ways (e.g., by knowing how bright something really is, versus how bright it looks). That gives us the answer, and the uncertainty in this is no more than a few percent.

The bigger question (so to speak) is the size of all there is, not just what we can see. The point is that we can only see things that are close enough for their light to get to us during those 13.7 billion years. However, a lot more could be out there that we haven't seen. The cosmos (i.e., all there is) seems to be at least tens of times larger than what we can see, and could well be infinite. We simply don't know at this stage, and may never know because there are only tiny observational differences between a very large cosmos and an infinite cosmos.

If the cosmos is infinite, this might answer your first question. Maybe the conditions to produce stars, planets, and life are very rare. For example, maybe you need just the right type of expansion, or values of fundamental constants. If the universe is infinite and in different volumes there are exotic things like different strengths of gravity or simply different evolutions, then with an infinite number of tries one would indeed expect that stars, planets, and life would appear in some cases. Only in those places would creatures such as ourselves appear to wonder why we exist, so that would not be surprising. This is called the anthropic principle: life only exists where life can exist (not a profound statement). I'm not saying that this is the definite answer, but it is one possibility.

Thanks for your questions!

Cole Miller

44. I am very interested in astronomy, especially the newer theories of quantum theory and dark matter and energy. This leads me to my question. I understand that the universe is expanding and that there are three possible outcomes. It could either expand forever, come to a dead stop, or begin to contract. My understanding is that the outcome of the universe depends on the amount of matter in the universe. Now as of now my knowledge is that there is no where near enough matter to stop the expanding, but that if there was a matter we could not see, it might cause enough inner pull to stop the contraction and begin the crunch. This leads to dark matter and that this mysterious substance might cause it. But my understanding is that we cannot measure dark matter or its effects, so how could it affect the universe enough to begin the big crunch?

Allow me to rephrase your interesting question: Is Dark Matter detectable and its effects measurable? Can it be the cause for a Big Crunch in the distant future?

Dark Matter was actually postulated in order to explain Galactic Rotation Curves. They were inferred to form a halo around visible galactic disks but were "Dark" to observation. Hence they are known to interact only gravitationally and their effects are observed on galactic or larger scales. Now assuming Dark Matter forms a significant fraction of the mass in the universe, if the total mass density exceeds the critical value for the universe to be mass dominated, theoretically it is possible for the universe to contract and undergo a Big Crunch in the very distant future. However the universe is presently undergoing accelerated expansion which is attributed to a mysterious component known as Dark Energy. Dark Energy is a hypothetical form of energy that has a strong negative pressure and is postulated to cause the observed expansion. Present understanding leads us to believe that the universe may continue to expand forever, however in time we might get a definitive answer to this problem.

Thanks for your question,

Sidharth Kumar

45. How do astronomers know the age of the universe?

Well, David, that is a good question! It's not exactly my area of study (I deal more with questions like "What did I see over in that part of the sky last night?) so I had to look up the answer.

The answer to "what is the age of the universe?" is not easy to find because there are many webpages that are pretty old and based on out of date data. But the most recent consensus seems to be that the Universe is about 13.7 billion years old give or take a couple of million years.

The "how do we know" is a bit easier. We make lots of observations and calculations.

We can observe: -- The ratios of various radioactive materials and by understanding their decay rates we can calculate how long that clump of material has been around.

-- Stars in clusters... We have a pretty good but not perfect understanding of how stars evolve. Higher mass stars evolve/die much more quickly than lower mass stars, so we can look at what stars are left in a cluster. Older clusters will only have their lower mass stars remaining while younger clusters will have some of their larger stars as well. There are clusters that have stars with only 0.7 solar masses implying that the clusters are between 11-18 billion years old.

-- cosmological redshift of galaxies and calculate Hubble's Constant. From this we can extrapolate the Age of the Universe. Yeah, this one is a bit tough to understand...

So over the years, as we've gotten better observations, we can refine the calculations.

Some webpages that explain the how we know in greater detail...

http://map.gsfc.nasa.gov/m_uni/uni_101age.html

http://imagine.gsfc.nasa.gov/docs/science/mysteries_l1/age.html

http://imagine.gsfc.nasa.gov/docs/features/exhibit/tenyear/age.html

http://ganymede.nmsu.edu/astro/a110labs/labmanual/node18.html

http://www.astro.ucla.edu/~wright/age.html

Clear Skies!

Elizabeth Warner

46. my second question has to do with dark energy. Basicly I wanted to know how it has so much energy? My third question has to do with dark energy as well. Since the discovory some five years ago have you noticed a fluctuation or a change in the pattern of how it is accelerating?

Questions about dark energy are among the most fundamental in astrophysics, and not a lot is known. To put it simply, people don't know where it comes from or why it has such a strong effect that it is currently dominating the expansion of the universe. Particle physicists hope that if they have a more complete understanding of fundamental particles and forces, then an answer to the puzzle of dark energy will be included in that understanding.

In the meantime, different observations are being planned that will try to characterize better how dark energy behaves, and thus answer your second question. We haven't seen any difference in the last five years; five years is so short compared to the roughly 14 billion year age of the universe that this is not expected. What we *can* do is do observe various epochs in the universe (i.e., different redshifts, if Mr. Edwards covered that) and try to measure precisely how the universe has expanded as well as how matter has come together to form clusters of galaxies. It is hoped that in the coming decades, observations using X-rays, visible light, and even gravitational waves will clear up how dark energy operates, and maybe that will give important clues about why it exists at all. Pretty mysterious; maybe you can help solve the problems!

Thanks for the questions,

Cole Miller

47. Recently, we have been discussing a little bit about dark energy and multiverses. What I don't understand is how there are multiverses. Is there just a giant gap of nothing between each universe? How can you tell where the edge of the universe is?

Hi. This is a really good question that shows one of the places where it is hardest to visualize things. You may want to check out any previous questions on ``What is the universe expanding into?'' or similar topics since they touch on many of the same ideas. They don't answer your question exactly, but they may help you think about these topics better.

For starters, we do not know for certain if there are other universes (a multiverse). Various theories of cosmology (some of the string and M theories for example) allow for or may even require multiple universes. However, since these theories are still far from certain, and we cannot observe other universes, this is still a very speculative area, but a multiverse has by no means been ruled out.

The problem is that there is no empty space that the universe is expanding into. Space, time, matter, and energy are all part of the universe. So while the universe is getting bigger, it is not expanding into something else. A consequence of this is that there isn't actually any edge of the universe. A good way to think of it is to imagine the surface of a sphere (like the Earth). If you are restricted to the surface of a sphere (as we mostly are to the surface of the earth) it doesn't make sense to talk about the edge because there is no edge. Just because the sphere is getting bigger doesn't change the lack of an edge. It is important not to take this picture too far since the sphere does expand into the space around it while the universe is just getting bigger itself.

So these (hypothetical at the moment) other universes are not separated from our universe by empty space: any empty space must be part of one universe or another. Different universes are completely separate from each other. If there was empty space between different ``universes'', given enough time, one could travel from one ``universe'' to the other, but this would mean that the two ``universes'' and the empty space between them are not actually different universes. They would just be far away parts of the same universe (and the empty space between them would also be part of that universe). With different universes in a multiverse, there is no way for anything to travel from one universe to another.

Imagine two sheets of graph paper with an x and y axis drawn on each of them. Each sheet has a grid and axes of its own. This grid represents two spatial dimensions. When you draw a line on one sheet of paper, the line can only be on the grid (so it is restricted to the two dimensions of the grid). If you drawn a line on one of the sheets, there is no way to extend the line to the other sheet while staying on your grid (and the sheet of paper). This is because each sheet of paper has its own x and y coordinates which have no relation to the x and y coordinates on the other sheet of paper. It is the same general idea for two separate universes, only now instead of two dimensions, you have 3 dimensions (or more) of space and one dimension of time.

Just to make things a little more interesting, the different universe may not even have the same values for fundamental constants (the strength of gravity and so on) or even the same number of spatial dimensions. So not only are alternative universe completely separate from ours, but they also could behave under very different physics than ours.

John C. Vernaleo

48. We have been talking about the universe and how it started lately. I understand that the early universe was very hot. I was wondering why it was so hot, and if we know how hot it actually was.

Hi. Good question. For the first part of your questions, why was the early universe so hot, there is a fairly clear answer: the early universe was so hot because it was so small and dense. When you compress something (gas in a cylinder for example), it gets hot. Since all the matter and energy that is around today was initially compressed in a very small space in the past, it was much hotter in the past than it is now.

The second part of your question ('do we know how hot it was') has a slightly more detailed. We do know how hot it was. Based on our current understanding of physics, we cannot know anything before the Planck time ($t=5.39121 x 10^{-44} s$). This is a fundamental limit set by Quantum Mechanics. At the Planck time, the temperature of the universe has to be the Planck Temperature, $T=1.41679 x 10^{32} K$. This is also just a fundamental limit imposed by Quantum Mechanics. On a side note, one can figure out these Planck values by carefully combining fundamental constants such as the Gravitational constant G, the speed of light c, and Planck's constant so that the units work out to the desired quantity.

From there, the Universe expands and cools rapidly. As long as we know the density and expansion rate of the universe, we can work out what the temperature is at different times. At the time of last scattering (this is the time where the Cosmic Microwave background radiation comes from, or about 300,000 years after the Big Bang), the average temperature of the universe was 3000K. The current temperature of the universe (ignoring all the little things like stars, galaxies, and us) is the temperature of the Cosmic Microwave Background (CMB), 2.73K. It is also possible to work backwards from the current measured value of the CMB temperature to previous values.

John C. Vernaleo

49. Just a few minutes ago, Mr. Edwards was explaining how the universe is expanding at quite the pace. We even discussed how there could be a multiverse and that another universe could be pulling on ours with its gravitational pull. Do you think this is possible? Is there any evidence? It was mind boggling to think that all these universes could have different physics than ours and possibly different equations like 1 + 3 = chair!

I think it's certainly possible that there are other universes, perhaps even an infinite number. Sometimes when people talk about other universes it has to do more with mathematics or philosophy than science, and sometimes other universes have been used as a theory to explain certain physical observations. However, there are other theories out there that work just as well or better in explaining the scientific data. There is presently no direct scientific evidence for other universes. My opinion is that it will be a very long time before scientists are able to devise an experiment to know if such things exist.

David Rupke

50. Last semester we took a trip to the planetarium at CSI. We watched a show that talked about the possibility on having multiple universes. What do you know about this subject, and what is your personal opinion on the possibility?

well, I personally don't know much about it, my personal science deals with galaxies and things a little bigger and a little larger perhaps. But I have also thought about this often, and as science progresses ideas about this have changed over the years.

Oddly, by definition, Universe means everything, so linguistically such a question doesn't seem to make sense. Maybe somewhat similar as asking where is the end of the circle? However, mathematically there are possibilities of "other" universes (but again, by the definition of universe, we cannot communicate with it). However, language is also not standing still. A new word, multiverse, has been coined to cover this problem. So, in a multiverse we can have many parallel universes.

And as with many things these days, you can find wonderful opionions on the internet with your favorite search engine. You may also encounter Brian Greene's name, who in addition to a wonderfully clear show on PBS has written a number of books in which this topic comes up. Max Tegmark is another name you will probably encounter. It's a fascinating topic, and a good question!

But be aware, the big World Wide Web has many snags, e.g. look at http://theflatearthsociety.org/forum/index.php?topic=11211.0 if you would go the other way and look for flat earth.

Peter Teuben

51. Today I would like to ask you about the big bang. My understanding about the big bang is that when it happened both matter and its opposite antimatter were created. But shouldn't the universe have created an equal amount antimatter and matter? We know this is not true because i am speaking to you, but what set off the unequal production of our matter over our opposite matter?

You're absolutely right! Experiments show for every particle of matter created there is a corresponding particle of anti-matter. If this same rule applied at the Big Bang, then where is the anti-matter?

First a little background on energy and matter vs. anti-matter. Matter and anti-matter are the opposite of each other, every matter particle has a corresponding particle of anti-matter. To preserve various conservation laws, the formation of a particle *must* be accompanied by the creation of its anti-particle. During this process *a lot* of energy becomes locked up in the particles' mass (from Einstein's equation, E=mc^2, where E is energy, m is the mass involved, and c is the speed of light, which, as you've learned, is a REALLY big number!). Invariably, the interaction of a particle with it's anti-particle results in the annihilation of both and a release of the energy that was "trapped" in their combined masses (on 'Star Trek' they use the energy released in this annihilation as a power source).

Right after the Big Bang *very close* to equal amounts of matter and anti-matter were produced. For every billion (10^9) particles of anti-matter, there was a billion and one particles of matter. Everything we see in the universe, from you and me to the farthest star is made of up of those one in a billion extra particles of matter. To give you an idea of how small the difference was, imagine two people racing from New York to San Francisco. The amount by which matter dominated anti-matter is the same as the winner of that 3000 *mile* race beating their opponent by less than half an *inch*.

So why is everything matter instead of anti-matter? The short answer to is we don't know. All of the experiments done until now have only been able to probe the lowest energies. However, things right after the Big Bang were very different, and involved much higher energies than anything we can recreate in a lab. At these energies, conservation laws as we understand them may no longer apply.

For an example of this, consider mass and energy. In chemical reactions, mass and energy are conserved as two separate, independent quantities, but when you start thinking about nuclear reactions, mass and energy can no longer be treated independently. Mass is now a measure of the energy "trapped" in the atomic nuclei. During these reactions, matter can convert back into energy via good olde E=mc^2. This is how nuclear power plants generate their energy (through fission), and how nuclear fusion keeps the Sun shining.

Exactly what conversation laws changed right after the Big Bang to allow matter to "win"? That remains to be seen.

Mia Bovill

52. My question has to do with the size of the universe and what could possibly be outside it (if there is an "edge" so to speak). We just finished talking in class about how theoretically if you could stand on the edge (whatever that is) of the universe and throw a baseball outward, what would it go into, or where? Mr. E told us about several theories that had to do with the fabric of time and other possible universes outside of ours but possibly effecting ours. What is theorized to be outside of our universe or between all the universes? Have astronomers begun to fathom the size of our universe and what are some other possibilities of what could be outside of it?

There is no edge to the universe, in the same way there is no edge to the surface of the Earth. Imagine a two dimensional being, let's call him Bob, moving on the surface of a balloon. The surface on which Bob moves is finite, but has no edges, and Bob can never throw something off of the surface, because in his world there are only the two dimensions of the surface of the balloon. You, me, the Earth, solar system, galaxy etc, all move in 3 dimensions of space and one of time, spacetime. Spacetime is to us, what the 2-D surface of the balloon is to Bob, we can never leave it, catapult anything out of it, or reach the edge of it.

Just like the 2-D surface of the balloon was embedded in 3-D space, spacetime may be embedded in additional dimensions that we can't observe. Within this higher dimensional structure could be other universes completely separate from our own with their own characteristics and physical laws. For a bit more on other universes see questions 13 and 22 in the cosmology section of the archive.

When astronomers talk about the size of the universe, they are usually referring to the size of the observable universe.

Nothing travels faster than light, and the speed of light is finite; so as we look further away we look further back in time. For instance, if an alien civilization 7 billion light years away were to look at the Milky Way right now, our Sun wouldn't be there, because the light the aliens are seeing left our galaxy 7 billion years ago, before the Sun was born. Conversely, we would see their galaxy as it was 7 billion years ago, before their sun existed!

If the universe was infinite old we would be able to see infinitely far, limited only by the ability of our telescopes to collect the light. However, the universe is about 13.7 billion light years old, so light from an object more than 13.7 billion light years away won't have had enough time to reach the Milky Way. The observable universe is a sphere centered on the Earth with a radius of 13.7 billion light years. For the alien civilization, their observable universe would be a sphere centered on their own homeworld 13.7 billion light years in diameter.

Mia Bovill

Send me to:

Cole's home page

astro home page